chike_eze
Thanks Received: 94
Atticus Finch
Atticus Finch
 
Posts: 279
Joined: January 22nd, 2011
 
 
trophy
Most Thanked
 

Q3 - A recent study proves that

by chike_eze Sun Jun 19, 2011 9:16 pm

This is an example of a question where I was caught over-thinking.
I took the #48 test untimed, and I missed this question.

I narrowed it down to A and C, and chose A. I now see why C is correct. I also see that B is irrelevant, D weakens, and E is out of scope. But I need some help explaining why A is incorrect.
 
chike_eze
Thanks Received: 94
Atticus Finch
Atticus Finch
 
Posts: 279
Joined: January 22nd, 2011
 
 
trophy
Most Thanked
 

Re: Q3 - A recent study proves that

by chike_eze Sun Jun 19, 2011 9:48 pm

I think I may have answered my own question. Please provide feedback either way.

(A) is wrong because it is incomplete. In this answer choice, we are told that the subjects claim that they were watched most of the time. But, we are not told whether their claim (about being watched most of the time) is actually true. So, even though they claimed that they were watched most of time, it is possible that they were wrong (or right) most (or some) of the time. We just don't know.

In my opinion, it neither strengthens nor weakens the argument.
Thoughts??
User avatar
 
bbirdwell
Thanks Received: 864
Atticus Finch
Atticus Finch
 
Posts: 803
Joined: April 16th, 2009
 
 
 

Re: Q3 - Sixth Sense

by bbirdwell Wed Jun 22, 2011 12:40 am

Sure, I think that's a fair assessment.

If anything, at a stretch, it could weaken. What if, since they were in front of a window, 99/100 people said "I'm being watched"?

All of a sudden it's not much of a study.
I host free online workshop/Q&A sessions called Zen and the Art of LSAT. You can find upcoming dates here: http://www.manhattanlsat.com/zen-and-the-art.cfm
 
patmichaelsmith
Thanks Received: 1
Forum Guests
 
Posts: 6
Joined: April 16th, 2012
 
 
 

Re: Q3 - A recent study proves that

by patmichaelsmith Mon May 21, 2012 2:16 pm

The conclusion of this argument is as follows:

"A recent study proves that at least some people possess an independent 'sixth sense' that allows them to detect whether someone is watching them."

Because the recent study is so central to the conclusion, wouldn't an answer choice backing up another study be out of scope as it has no influence on the conclusion?

I know this is a strengthen question where answer choices can and do contain extraneous information, but to me, choice C doesn't help support the conclusion that the first study is what's responsible for proving an independent sixth sense.

Can anybody help me clear this up?
 
timmydoeslsat
Thanks Received: 887
Atticus Finch
Atticus Finch
 
Posts: 1136
Joined: June 20th, 2011
 
 
trophy
Most Thanked
trophy
First Responder
 

Re: Q3 - A recent study proves that

by timmydoeslsat Mon May 21, 2012 6:44 pm

The question stem asks us to strengthen the conclusion reached from the first study. So our conclusion is really that at least some people contain that sixth sense.
 
tzyc
Thanks Received: 0
Atticus Finch
Atticus Finch
 
Posts: 323
Joined: May 27th, 2012
 
 
trophy
Most Thankful
 

Re: Q3 - A recent study proves that

by tzyc Wed Aug 15, 2012 1:29 am

Is (C) correct because even they were watched on video monitor (through screen/different ways) they could answer well as the first experiment, and that supports/adds evidence to the conclusion?
User avatar
 
ManhattanPrepLSAT1
Thanks Received: 1909
Atticus Finch
Atticus Finch
 
Posts: 2851
Joined: October 07th, 2009
 
This post thanked 2 times.
 
 

Re: Q3 - A recent study proves that

by ManhattanPrepLSAT1 Thu Aug 16, 2012 4:01 pm

Exactly right tz_strawberry! If the subjects could correctly say when they were being watched by means of a video camera that adds a second form in which the subjects could correctly identify whether or not they were being observed.

Answer choice (C) helps to eliminate the possibility (however unlikely) that the subjects through some sort of reflection or whatever could see the observer through the window!
 
eapetrilli
Thanks Received: 5
Forum Guests
 
Posts: 11
Joined: August 06th, 2012
 
 
 

Re: Q3 - A recent study proves that

by eapetrilli Fri Sep 07, 2012 1:45 pm

I correctly chose C but had a different reason. I thought that them being watched by camera strengthed the argument because it strengthened the likelihood that the experiment had been adequately controlled. Maybe the subjects had some tricks up their sleeve that only a camera could detect, but C rules out that possibility because of that duplicated result in a more controlled setting.
 
pewals13
Thanks Received: 15
Elle Woods
Elle Woods
 
Posts: 85
Joined: May 25th, 2013
 
 
 

Re: Q3 - A recent study proves that

by pewals13 Thu Mar 24, 2016 11:38 am

I think that's perfectly fine, the duplication under slightly different circumstances decreases the chance that a variable unique to the original test conditions was responsible for the result.
 
QingyiY938
Thanks Received: 0
Vinny Gambini
Vinny Gambini
 
Posts: 5
Joined: May 21st, 2024
 
 
 

Re: Q3 - A recent study proves that

by QingyiY938 Tue May 21, 2024 4:26 pm

I was also between A and C. I ultimately chose A because the conclusion was saying that people with 6 sense could tell when people were watching and the study gave the support that 60 percent of the time they could tell if ppl were watching or not. I think I kind of read too deeply into the stimulus and thought A gave more support for the "watch" portion. I liked C but chose A because of that reason. However, reading A again, I think it actually weakens the argument because it suggests that maybe these people were guessing and don't actually have a 6th sense.